You are on page 1of 79

In a developing country, since 1960, the number of millionaires in an urban city has been

increasing by 100% every 10 years while the number of millionaires of a rural town has
been increasing by 200% every 10 years. In the year 2000, the sum of the number of
millionaires in both the places was 337. What was the absolute difference between the
number of millionaires in the city and the town in the year 1960?

1) 15

2) 16

3) 1

4) 9

5) 0
Solution:
Let number of millionaires in the city and town in 1960 be x and y, respectively.

An increase of 100% in x will make it 2x, while an increase of 200% in y will make it 3y,
and so on.

In 2000, the number of millionaires in the city = 2 2 2 2 x = 16x

And, the number of millionaires in the town = 3 3 3 3 y = 81y

Sum of millionaires in city and town = 16x + 81y = 337


The only set of integer values that satisfies this equation is x = 16, y = 1.

Difference in number of millionaires in 1960 in the city and the town = 16 1 = 15

Hence, option 1.


2.
3
Marks
How many six-digit numbers are there such that the 3
rd
digit is the square of the
1
st
digit and the 4
th
digit is the square of the 2
nd
digit?

1) 256

2) 1000

3) 900

4) 6400

5) 1200
Solution:
We need to find the number which is of the form aba
2
b
2
cd where a, b, c and d
can take any value from 0 to 9.

a and b cannot exceed 3 as the square of the two numbers then would exceed 9,
which is not possible.

a can have 3 values (1, 2 or 3) and b can have 4 values (0, 1, 2 or 3). c and d
can have 10 values each.

Thus total count of numbers that satisfy the given conditions is 3 4 10 10 =
1200


Hence, option 5.


3.
3
Marks


In the above figure, AB is the diameter and CD is parallel to AB. Also BC = AD
= 2. AB = 8. What is the length of CD?

1) 5

2) 6

3) 7

4) 4

5) None of these
Solution:


From the figure, O is the center of a circle and CE is perpendicular to OB.

Let EB = x

OE = 4 x

Applying Pythagoras theorem to OCE, we get,

4
2
= (4 x)
2
+ CE
2


CE
2
= 8x x
2


Applying Pythagoras theorem to CEB, we get,

2
2
= x
2
+ 8x x
2


x = 1/2

But CD = AB 2x = 8 1 = 7

Hence, option 3.


4.
3
Marks
Each question is followed by two statements, A and B. Answer each question
using the following instructions:
Mark (1) if the question can be answered by using statement A alone but not by
using statement B alone.
Mark (2) if the question can be answered by using statement B alone but not by
using statement A alone.
Mark (3) if the question can be answered by using either statement alone.
Mark (4) if the question can be answered by using both the statements together
but not by either of the statements alone.
Mark (5) if the question cannot be answered on the basis of the two statements.
If a and b are integers, what is the value of a + b?
A. LCM of a and b is 34.
B. One of the two numbers is 17.


1) 1

2) 2

3) 3

4) 4

5) 5
Solution:
Using Statement A alone:

LCM(a, b) = 34

It is evident that the values of a and b are not unique.

Thus, the question cannot be answered using statement A alone.

Using Statement B alone:

There is no information on the second number.

Thus, the question cannot be answer using statement B alone.

Using both the statements together:

LCM(a, b) = 34 and one of these two numbers is 17.

The other number could be 2 or 34.

Hence, the sum a + b can still not be determined.

Thus, the question cannot be answered on the basis of the two statements.

Hence, option 5.

5.
Equation of a pair of line passing through the origin is 2x
2
+ xy y
2
= 0. What
3
Marks
are the separate equations of the lines?

1) 2x + y = 0 and x y = 0

2) 2x y = 0 and x y = 0

3) 2x + y = 0 and x + y = 0

4) 2x y = 0 and x + y = 0

5) None of these
Solution:
2x
2
+ xy y
2
= 0 is a second degree equation that represents a pair of straight
lines passing through the origin.

2x
2
+ 2xy xy y
2
= 0

2x(x + y) y(x + y) = 0

2x y = 0 and x + y = 0 are the separate equations of the line.

Hence, option 4.


6.
3
Mark
s


1) 2

2) 3.6

3) 4.2

4) 5

5) 4
Solution:



the line 4x + 3y = 14.





Hence, option 5.


7.
3
Marks
Each question is followed by two statements, A and B. Answer each question
using the following instructions:
Mark (1) if the question can be answered by using statement A alone but not by
using statement B alone.
Mark (2) if the question can be answered by using statement B alone but not by
using statement A alone.
Mark (3) if the question can be answered by using either statement alone.
Mark (4) if the question can be answered by using both the statements together
but not by either of the statements alone.
Mark (5) if the question cannot be answered on the basis of the two statements.
Is 2x = y?

A.

B. (2x 9)
2
= (y 9)
2


1) 1

2) 2

3) 3

4) 4

5) 5
Solution:
Consider statement A alone:



Hence, (2x + y)
2
= 8xy

Thus, (2x y)
2
= 0

Or 2x = y

Hence statement A alone is sufficient to answer the question.

Consider Statement B alone:

(2x 9)
2
= (y 9)
2


Thus, 2x 9 = y 9 or 2x 9 = 9 y

Hence, 2x = y or 2x = 18 y

Hence, no definite conclusion is possible.

Hence, we cannot answer the question using statement B alone.

Hence, option 1.


8.
3
Marks
In Dexters laboratory there is a specimen of bacteria that doubles its
number every minute. On certain day, the number of bacteria at 6.10 pm was
1048576. What was the number of bacteria present at 6.00 pm the same evening?

1) 9216

2) 512

3) 1024

4) 32

5) None of these
Solution:
Let the number of bacteria present at 6.00 pm be n.

Thus at 6.10 pm it will be,

n(2)
10
= 1048576

n = 1024

Hence, option 3.

Group Question


Answer the following questions based on the information given below.


The following graph gives the sales revenue of five companies in the years from
2000 to 2004.



9.
3
Marks
The maximum absolute decrease in the sales revenue of CD occurred between
which of the following years?

1) 2000 and 2001

2) 2001 and 2002

3) 2002 and 2003

4) 2003 and 2004

5) 2001 and 2004
Solution:
Note that the absolute decrease and not the percentage decrease is required.

The sales revenue for CD has decreased only twice, i.e. between 2001 and
2002 and between 2002 and 2003. Also, the sales revenue for CD in 2004 is
more than the sales revenue in 2001.

Hence, options 1, 3 and 4 can be eliminated.

From the year 2001 to 2002, absolute decrease registered for CD = 75 50 =
25

From 2002 to 2003, the absolute decrease = 50 0 = 50

The absolute decrease in sales value of CD is maximum from 2002 to 2003.

Hence, option 3.


10.
3
Marks
The absolute difference between the sales revenue of GH and IJ was the least
in the year

1) 2000

2) 2001

3) 2002

4) 2003

5) 2004
Solution:
The absolute difference between the sales revenue of GH and IJ for each year
is given below.

For 2000 = 100 50 = 50

For 2001 = 125 75 = 50

For 2002 = 100 100 = 0

For 2003 = 125 75 = 50

For 2004 = 100 75 = 25

The absolute difference between the sales revenue of GH and IJ was the
least in the year 2002.

Hence, option 3.


11.
3
Marks
In which of the following years was the ratio of the sales revenue of GH to
that of EF the maximum?

1) 2000

2) 2001

3) 2002

4) 2003

5) 2004
Solution:
The ratio of the sales revenue of GH to that of EF for each year is shown
below.

For 2000 : 100/25 = 4

For 2001 : 125/50 = 2.5

For 2002 : 100/75 = 1.33

For 2003 : 75/50 = 1.5

For 2004 : 75/75 = 1

The ratio of the sales revenue of GH to that of EF was maximum in the year
2000.

Hence, option 1.

Note: This question can be answered by direct observation as well. It is
evident from the bar chart that the ratio is the highest in 2000.


12.
3
Marks
A boy, standing on top of a building 25 m high, notices a bird at some distance at
an angle of elevation of 30. At the same instant, he also notices reflection of the
bird at an angle of depression of 45 in a small water pool, on the same level as
the floor of the building and exactly vertically below the bird. What is the distance
between the boy and the bird?

1)


2)


3)


4) 25

5) None of these
Solution:
Given data can be represented as shown in the following diagram.



Let AB be the building, E be the location of the bird and C be the point on the
pool on which the shadow of the bird is cast.

The measure of all four angles of the quadrilateral ABCD is 90, therefore, ABCD
is a rectangle.

AB = DC = 25 m

In right angled ADC, DAC = 45

ADC = 45

AD = DC = 25 m

In ADE, DAE = 30









Hence, option 2.


13.
3 Marks
The roots of the equation 4x
4
4x
3
72x
2
4x + 4 = 0 are

1)


2)


3)


4)


5)

Solution:
4x
4
4x
3
72x
2
4x + 4 = 0

Dividing by x
2
on both the sides











4(a
2
2) 4(a) 72 = 0

a
2
a 20 = 0

a = 5 or a = 4

When a = 5



x
2
5x + 1 = 0



When a = 4



x
2
+ 4x + 1 = 0





Hence, option 2.


14.
3
Marks
A piece of work when done by Amar, Akbar and Anthony respectively, costs Rs.
540, Rs.324 and Rs. 320 respectively. The daily wages of Amar, Akbar and
Anthony are Rs.36, Rs.18 and Rs.32 respectively. What will be the total cost when
Amar, Akbar and Anthony are working together?

1) 480

2) 378

3) 387

4) 487

5) None of these
Solution:






The fraction of job that will be completed in a day when all three are working
together is



4.5 days will be required to finish the job when all the three are working together.



Hence, option 3.


15.
3
Marks
Kishore owns a factory of plastic goods. His production cost comprises only the
cost of raw material and labour . The cost of raw material increases by 20% and
the labour cost also increases from 20% to 25% of the cost of raw material. By
how much percent should he reduce his consumption of raw material and labour
so that his production cost remains unchanged?

1) 20%

2) 15%

3) 17%

4) 25%

5) None of these
Solution:
Let the cost of raw materials be Rs. 100

Then the labour cost will be Rs. 20

Production cost = 120

Due the increase in cost of raw materials and labour cost the revised cost of raw
material and labour cost will be Rs. 120 and Rs. 30 respectively.

The new production cost will be Rs. 150

In order to bring back the production cost to Rs. 120 he should reduce the
consumption of raw materials and labour.

Percentage reduction in consumption of raw material and labour will be,



Hence, option 1.


16.
3
Marks
What is the number of sides of a polygon for which sum of interior angles is
5400?

1) 20

2) 32

3) 19

4) 18

5) None of these
Solution:
For an n sided polygon, there exist n vertices and therefore n interior angles.

As we know, for every interior angle, there exists an exterior angle and thus n
exterior angles exist.



Therefore, interior angle = 180 exterior angle





It is given that the sum of the interior angles = 5400

180(n 2) = 5400

n = 32

Hence, option 2.


17.
3
Mark
s
Consider the following statements







Which of the statements is/are incorrect?

1) II only

2) III only

3) I only

4) I and II

5) II and III
Solution:






Thus only statement II is incorrect.

Hence, option 1.


18.
3
Marks
Jai and Veeru are two masons who are given the job of building a compound wall
around a plot owned by Mr. Thakur. Jai working alone takes 8 hours more than
the time that both Jai and Veeru would take working together. Veeru working
alone takes 12.5 hours more than the time that both would take working
together. How many hours would Jai alone take to build the wall?

1) 10 hours

2) 18 hours

3) 12 hours

4) 16 hours

5) None of these
Solution:
Let the time taken to build the compound when both Jai and Veeru are working
together be x.

Time taken by Jai to complete the job alone is x + 8 and the time taken by Veeru
to complete the job alone is x + 12.5

Thus we get the following equation:



2x
2
+ 20.5x = x
2
+ 20.5x + 100

x
2
= 100

x = 10

Thus Jai will take 18 hours to complete the job alone.

Hence, option 2.


19.
3
Marks
Each question is followed by two statements, A and B. Answer each question
using the following instructions:
Mark (1) if the question can be answered by using statement A alone but not by
using statement B alone.
Mark (2) if the question can be answered by using statement B alone but not by
using statement A alone.
Mark (3) if the question can be answered by using either statement alone.
Mark (4) if the question can be answered by using both the statements together
but not by either of the statements alone.
Mark (5) if the question cannot be answered on the basis of the two statements.
Six friends Aziz, Vivek, Nipun, Arun, Sahil and Navnit are sitting around a
circular table such that Arun is two places to the left of Aziz. Who is opposite
Vivek?
A. Sahil is adjacent to Nipun and Vivek
B. Vivek is adjacent to Aziz and Sahil


1) 1

2) 2

3) 3

4) 4

5) 5
Solution:
The basic arrangement that can be obtained from the question is as shown below.



Using Statement A alone:



It is clear from the arrangement that either Arun or Aziz can be opposite Vivek.

Thus, the question cannot be answered using statement A alone.

Using Statement B alone:

If Vivek is adjacent to Aziz and Sahil, Vivek has to be between Aziz and Sahil.

The only way this is possible is if Vivek is adjacent to Aziz on his right and Sahil
is to the immediate right of Vivek.

Now, Arun is opposite to Vivek as shown in the figure below.



Thus, the question can be answered using statement B alone.

Thus, the question can be answered using statement B alone but not by using
statement A alone.

Hence, option 2.


20.
3
Marks
The difference between simple interest and compound interest on a particular
amount in 2 years at 30% is Rs.360. What is the principal?

1) Rs. 4000

2) Rs. 3500

3) Rs. 3000

4) Rs. 2500

5) None of these
Solution:
The difference between the simple and compound interest for two years is the
simple interest on the simple interest for one year.



i = 1200

Interest for the first year = Rs. 1200



Hence, option 1.


21.
3
Marks
n
th
term of an AP, consisting of only positive integers, is denoted by a
n
. It is
given that a
3
= 4 and a
N 2
= 9. What is the value of (a
N
+ 2N + 2)?

1) 29

2) 31

3) 33

4) 35

5) Cannot be determined
Solution:
Let common difference of the AP be d.

We have a
1
= a
3
2d = 4 2d

We also have a
1
= a
N 2
(N 3)d = 9 d(N 3) = 9 Nd + 3d

Equating these two values of a
1
, we get,

4 2d = 9 Nd + 3d, or (N 5) d = 5

Since both N and d are positive, we must have N 5 = 1 and d = 5 or N 5 = 5
and d = 1.

Thus, either N = 6 and d = 5 or N = 10 and d = 1.

In the first case, a
N
+ 2N + 2 = a
1
+ (N 1) d + 2N + 2 = 4 2 5 + 5 5 + 2 6
+ 2 = 33

In the second case, a
N
+ 2N + 2 = a
1
+ (N 1) d + 2N + 2 = 4 2 1 + 9 1 + 2
10 + 2 = 33

In either case, the given expression = 33

Hence, option 3.


22.
3
Marks
Each question is followed by two quantities, A and B. Answer each question
using the following instructions:
Mark (1) if quantity A is greater than quantity B.
Mark (2) if quantity B is greater than quantity A.
Mark (3) if the two quantities are equal.
Mark (4) if it is impossible to determine a relationship.
Mark (5) if the greater quantity cannot be determined but the two quantities are
definitely not equal.
In a new dictionary, the position of alphabets is changed so as to arrange the
words more concisely.

Let P be the old position of alphabets from 1 to 26, and let P
new
be the new
position.

P
new
is calculated using the function f(P), defined as follows:

f(P) = remainder of P quotient of P, when divided by 3. ... If none of them are
0.

= quotient of P, when divided by 3. ... If the remainder is
0.

f(P) is calculated for all the values of P (i.e. 1 to 26) and arranged in the
ascending order in a set A.

If f(P) for two values of P is the same, then the one with a higher quotient is
placed after the one with lower quotient, and if they have the same quotient, then
one with higher P is placed after the one with lower P.

Now, the new position of a letter P is defined as the position of f(P) in the set A.
A. Position of the word MAS in dictionary
B. Position of the word MAN in dictionary


1) 1

2) 2

3) 3

4) 4

5) 5
Solution:
Since the first two alphabets are the same, to find the order of the words, find the
location of S and N in the new dictionary.

P for S and N is 19 and 14 respectively.

f(14) = 2 4 = 8

and

f(19) = 1 6 = 6

f(19) < f(14)

Hence, the word MAS is placed before the word MAN in the new dictionary.

Position of the word MAN > Position of the word MAS

Hence, option 2.


23.
3
Marks
Each question is followed by two statements, A and B. Answer each question
using the following instructions:
Mark (1) if the question can be answered by using statement A alone but not by
using statement B alone.
Mark (2) if the question can be answered by using statement B alone but not by
using statement A alone.
Mark (3) if the question can be answered by using either statement alone.
Mark (4) if the question can be answered by using both the statements together
but not by either of the statements alone.
Mark (5) if the question cannot be answered on the basis of the two statements.
Is the 1
st
number in a series of 7 consecutive integers even?
A. The arithmetic mean of the numbers is 2.
B. Sum of the numbers is divisible by 11.


1) 1

2) 2

3) 3

4) 4

5) 5
Solution:
Using Statement A alone:

Since the series consists of 7 consecutive integers, it forms an A.P. consisting of
odd number of terms and the middle term of that A.P. is also the arithmetic mean
of the A.P.

Thus, the arithmetic mean of the 7 numbers is the 4
th
number which is 2.

Therefore, the 1
st
number in the series is 2 3 = 1.

Thus, the 1
st
number is odd.

Thus, the question can be answered using statement A alone.

Using Statement B alone:

Sum of the 7 numbers = (x 3) + (x 2) +( x 1) + x + (x + 1) + (x +2) + (x + 3)
= 7x

Thus, the sum of the given numbers is divisible by 7.

Now, it is known this sum is also divisible by 11 which means that x has to be a
multiple of 11.

But x can be odd or even which implies that the first term can also be odd or
even.

Thus, the question cannot be answered using statement B alone.

Thus, the question can be answered using statement A alone but not by using
statement B alone.

Hence, option 1.


24.
3
Marks
A train moving at 54 kmph crosses a man standing on a platform in 10 seconds.
What will be the time required by the train to cross a platform if the length of the
platform is equal to the length of the train?

1) 30 seconds

2) 50 seconds

3) 25 seconds

4) 20 seconds

5) 45 seconds
Solution:


Distance covered by the train to cross a man standing on the platform = Length of
the train

Distance = Speed Time

Distance covered = 15 10 = 150 m

So the length of the train is 150 m.

Length of the platform = Length of the train = 150 m

Time taken by the train to cross the platform = Total Distance covered (Length of
platform + Length of train)/Speed of train



Hence, option 4.


Alternatively,

As the length of the platform is equal to that of the train, the train will require
twice the time to cross it as it takes to cross the man.

Hence, option 4.


25.
3
Marks
There are 3 friends competing against each other in a race. Mohit beats Purohit in
a race of 1000 m by 50 m and Purohit beats Rohit in a race of 1200 m by 100 m.
By what margin (in m)would Mohit beat Rohit in a race of 1500 m? (Round off
the answer to closest integer.)

1) 190

2) 194

3) 197

4) 198

5) 199
Solution:
Let the speed of Mohit, Purohit and Rohit be m, p and r respectively.

In a race of 1000 m,





In a race of 1200 m,







Let Mohit beats Rohit by x in a race of 1500 m .





5225 = 6000 4x

4x = 775 or x = 193.75

Hence, option 2.


26.
3
Marks
What is the ratio of the right most digit preceding the zeros in the value of 20
53
to
the right most digit preceding the zeros in the value of 40
53
.

1) 1 : 2

2) 2 : 3

3) 2 : 1

4) 3 : 2

5) None of these
Solution:
20
53
= 2
53
10
53


The rightmost digit preceding zeros will be the unit's digit of 2
53


2
1
= 2, 2
2
= 4, 2
3
= 8, 2
4
= 16, 2
5
= 32 and so on.

Thus we can see that the units digit repeats after every 4
th
power.

53 = 52 + 1, where 52 is a multiple of 4.

The unit's digit of 2
53
is same as the unit's digit of 2
1
, which is 2.

Similarly we can show that the right most digit preceding the zeros in the value
of 40
53
is 4.

Hence, the ratio is 1 : 2

Hence, option 1.


27.
3
Marks
In the figure below, mABC = 40, mBAD = 140, mADE = 130 and DC =
DE. Find mCDE + mDCB.



1) 160

2) 170

3) 210

4) 150

5) None of these
Solution:
In quadrilateral ABED,

mABE + mBAD + mADE + mDEB = 360

mDEC = 50

Since DC = DE

mDEC = mDCE = 50

Now, in DCE,

mDCE + mDEC + mCDE = 180

mCDE = 80

Now, mDCE + mDCB = 180

mDCB = 130

mCDE + mDCB = 210

Hence, option 3.

28.
In class A, the ratio of the average age of boys to the average age of girls is 1 : 2
3
Marks
and the ratio of number of boys to the number of girls is 2 : 1. In class B, the ratio
of average age of boys to the average age of girls is 3 : 2 and the ratio of number
of boys to the number of girls is 1 : 2. The number of students in class A and class
B are equal. If the average age of class A is equal to the average age of class B,
then what is the ratio of the average age of girls of class A to the average age of
boys of class B?

1) 7 : 6

2) 6 : 5

3) 5 : 3

4) 2 : 3

5) 5 : 4
Solution:
For class A:

Let the average age of boys be x and the number of girls be y.

Average age of girls is 2x and the number of boys is 2y.



For class B:

Let z be the common variable for average age of boys and girls.

Average age of boys is 3z and average age of girls is 2z.

Since, the total number of students is equal in both classes, the number of boys is
y and the number of girls is 2y.



Now, since the average ages of both classes are equal





Ratio of average age of girls of class A to the average age of boys of class B is



Hence, option 1.


29.
3
In Vrindavan Housing Society the monthly society expenditure has a fixed and a
Marks variable component, such that the variable component depends on the number of
members in the society. If there are 50 members, then each member has to pay Rs.
110 and if there are 30 more members then each member has to pay Rs. 80. If
each member pays Rs. 130, then how many members are there in the society?

1) 45

2) 55

3) 40

4) 60

5) None of these
Solution:
It is given that, expenditure = F + V

From the condition given we can say that,

(110)(50) = F + 50k ...(i)

(80)(80) = F + 80k ...(ii)

On solving (i) and (ii) we get,

F = 4000 and k = 30

Now,

130x = 4000 + 30x, where x is the number of members

x = 40

Hence, option 3.

Group Question


Answer the following questions based on the information given below.


The graph below gives details of total exports as well as exports of herbal products for a
company.

The abscissa represents the total exports for a given year in Rs. Lakhs, while the
ordinate represents the exports of herbal products for that same year in Rs. Lakhs.



30.
3
Marks
For 2006, herbal products formed what percentage of the total exports?

1) 13.50%

2) 13.64%

3) 14.28%

4) 12.50%

5) 11.11%
Solution:
The abscissa and ordinate stand for the X-axis and Y-axis respectively.

Thus, figures on the X-axis stand for the total exports for a given year while
those on the Y-axis stand for the export of herbal products in that year.

Thus, in 2006, the total exports were Rs. 5,500 lakhs, whereas the exports of
herbal products were Rs. 750 lakhs.



Hence, option 2.


31.
3
Marks
The fall in herbal exports in 2007 from that in 2006 was nearly:

1) 13.65%

2) 12.78%

3) 13.33%

4) 86.66%

5) 90.25%
Solution:
In 2006, the herbal exports were Rs. 750 lakhs, whereas in 2007, they were Rs.
650 lakhs.



Hence, option 3.


32.
3
Mark
s
Over the given period, the herbal exports rose by nearly:

1) 300%

2) 30%

3) 10%

4) 18.65%

5) 77%
Solution:
In 2004, the herbal exports were Rs. 500 lakhs while at the end of the period, i.e. in
2007, they were Rs. 650 lakhs.



Hence, option 2.


33.
3
Marks
In how many ways can a committee of 4 people comprising at least 3 boys be
formed using a group of 5 boys and 6 girls?

1) 65

2) 34

3) 60

4) 46

5) None of these
Solution:
Atleast 3 boys means the committee can be formed by 3 boys and 1 girl or all 4
boys.

Case 1: 3 boys and 1 girl

From 5 boys and 6 girls they can be selected in
5
C
3

6
C
1
ways = 10 6 = 60
ways

Case 2: All 4 boys

They can be selected from 5 boys in
5
C
4
ways = 5 ways

Total number of ways = 60 + 5 = 65 ways

Hence, option 1.


34.
3
Marks
A, B and C invest Rs. 4000, Rs. 15000 and Rs. 5000 respectively to set up a new
business. A gets 20% of the total profit for running the business and the remaining
profit is divided among the three in proportion to their investments. If in a year, A
gets Rs. 3000 less than B and C together, the total profit for the year is

1) Rs. 5000

2) Rs. 6000

3) Rs. 7500

4) Rs. 9000

5) Rs. 10500
Solution:
Ratio of investments of A, B and C 4000 : 15000 : 5000 4 : 15 : 5

Let the profit for the year be Rs. x

Profit given to A for managing the business = 20% of x = 0.2x

80% of the total profit is divided between A, B and C in ratio 4 : 15 : 5

80% of total profit = 0.8x







A earns Rs. 3000 less than B and C.



x = 9000

Hence option 4.


35.
3
Marks
In PQR, a line is drawn from P to intersect the opposite side QR at S. What is m
QPS such that PR = RS and m RPQ = m PQR + 60?

1) 15

2) 30

3) 20

4) 25

5) 45
Solution:


m QPS = m RPQ m RPS

PR = RS

m QPS = m RPQ m PSR

Using exterior angle theorem, we get,

m PSR = m QPS + m PQS

m QPS = m RPQ (m QPS + m PQS)

m PQS = m PQR

2(m QPS) = m RPQ m PQR

2(m QPS) = 60

m QPS = 30

Hence, option 2.


36.
3
Marks
A field is in the form of a circle of radius 100 m; the field is covered with grass
except for an area in the form of a circle concentric with the field, and of radius 50
m. A parachutist is falling towards the field because his parachute has failed to
open; he knows that he will die unless he falls on grass. He may fall anywhere on
the field. What is the probability that he will live?

1)


2)


3)


4)


5) 1
Solution:
The probability that the parachutist will fall at any point on the field is equal to the
probability that he will fall at any other point on the field.

Thus, the probability that he will fall on grass (and hence live) is equal to the area
of the grassy portion of the field divided by the total area of the field.

The area (in sq. m.) of the grassy portion = [100
2
50
2
]

The area (in sq. m.) of the entire field = [100
2
]



Hence, option 3.


37.
3
Marks
Gayathri and Savithri sell apples. Savithri sells two apples for one rupee. The
apples that Gayathri sells are a bit smaller; she sells three apples for one rupee. At
a certain moment, when both ladies have the same amount of apples left, Gayathri
is called away. She asks her neighbour to take care of her goods. To make
everything not too complicated, Savithri simply puts all apples together, and starts
selling five apples for Rs. 2. When Gayathri returns the next day, all apples have
been sold. But when they start dividing the money, there appears to be a shortage
of Rs. 7. Supposing they divide the amount equally, how much does Savithri lose
in this deal?

1) 12

2) 21

3) 0

4) 25

5) Cannot be determined
Solution:
The big pile of apples contains the same number of large apples of half a rupee
each (from Savithri), as smaller apples of one third a rupee each (from Gayathri).



The apples should be sold at Rs. (5/12) each.

But the apples were sold for Rs. (2/5) each (5 apples for Rs. 2).

Each apple is sold at Rs. (1/60) less.

The total shortage is Rs. 7,

After combining there were 7 60 = 420 apples.



They divided the money equally amongst themselves, each of them got Rs. 84.

If Savithri would have sold the apples herself, she would have received Rs. 105
for 210 apples.

Savithri loses Rs. 21 in this deal.

Hence, option 2.


38.
3
Marks
A number which reads the same when read forward and backward is called a
palindrome.
How many four-digit numbers are palindromes?

1) 81

2) 90

3) 99

4) 100

5) 115
Solution:
A 4-digit palindrome will be of the form abba where a > 0 and b can take integer
values from 0 to 9.

a can take 9 values and b can take 10 values.

The total number of 4-digit palindromes is 9 10 = 90

Hence, option 2.


39.
3
Marks
Seven lotuses, sixteen roses and nine lilies cost Rs. 338. Four lotuses, six roses
and seven lilies cost Rs. 169. What is the cost of nine lotuses, thirty roses and six
lilies?

1) Rs. 169

2) Rs. 507

3) Rs. 496

4) Rs. 570

5) Cannot be determined
Solution:
Let the price of a lotus, a rose and a lily be Rs. x, y and z respectively.

7x + 16y + 9z = 338 ...(i)

and

4x + 6y + 7z = 169 ...(ii)

Subtracting (ii) from (i) and multiplying the resulting equation by 3, we get

9x + 30y + 6z = 507

So the cost of 9 lotuses, 30 roses and 6 lilies is Rs. 507.

Hence, option 2.


40.
3
Marks
In a survey conducted 70% watched CNN, 75% watched BBC, 20% watched
neither of the channels and 325 people watched both the channels. How many
people were surveyed?

1) 500

2) 382

3) 50

4) 492

5) 491
Solution:

Let x be the total number of people surveyed.

From the figure above,

0.7x 325 + 325 + 0.75x 325 + 0.2x = x

x = 500

Hence, option 1.


41.
3
Marks
The length of the side of a cubical box is equal to twice the length of the longest
rod that can be placed in a rectangular box of dimensions 3 cm 4 cm 12 cm.
What is the maximum number of spheres of diameter 32.5 mm that can be kept in
the cubical box?

1) 64

2) 512

3) 216

4) 1000

5) 324
Solution:
The length of the longest rod that can fit in the cuboid

= The diagonal of the cuboid



Side of the cubical box = 2 13 = 26 cm


Along one face of the cube 8 8 sheres can be fitted as shown in the figure
given below.



Also, there will be 8 such vertical slabs in the cube.

The total number of spheres that can be fitted in the cube = 8 8 8 = 512

Hence, option 2.


42.
3
Marks
If m and n are two natural numbers, ratio of m to n is x and ratio of n to m is y,
then (x + y) will be

1) Less than 1

2) Greater than 1

3) Equal to 1

4) Depends on m and n

5) Data insufficient
Solution:


x + y > 1



Even then x + y > 1

x + y is always greater than 1.

Hence, option 2.


43.
3
Marks
There are three identical circles of radii 10 units touching each other. What is
circumference of the biggest circle that can fit in the gap formed by the
intersection of all three circles and touching all three of them?

1)


2)


3)


4)


5) None of these
Solution:


Let the radius of a smaller circle be r.

ABC is an equilateral triangle.





the centroid divides the median of a triangle in the ratio 2 : 1,






Circumference = 2r




Hence, option 2.


44.
3
Marks
On a busy Monday morning a certain number of cyclists started from Borivali
towards Churchgate. At the Malad junction, half of these cyclists entered Malad
while 1/5 of the remaining cyclists joined the cyclists going towards Chuchgate.
The same thing happened at Andheri, Santacruz, Bandra and Dadar. No cyclist
joined or left the group thereafter. Finally 243 cyclists reached Churchgate. How
many cyclists started from Borivali?

1) 3125

2) 1875

3) 2225

4) 2875

5) None of these
Solution:
There are 243 cyclists at Churchgate. So 243 cyclists left Dadar.

Let there be x cyclists before Dadar.

Since half of the cyclists left, there were x/2 cyclists remaining.

These were joined by 1/5 of remaining cyclists.



This has to be equal to 243.

x = 405

Using the same logic, we have the number of cyclists before Bandra to be 675,
before Santacruz to be 1125, before Andheri to be 1875 and before Malad to be
3125.

Thus total cyclists leaving Borivali is 3125.

Hence, option 1.


45.
3
Marks
Ram invested Rs. 10,000 each in two bank schemes. In scheme A he earns an
interest of 10% p.a., compounded semi-annually, while in scheme B he gets a
simple interest of 6% every six months. What is the difference in the interest that
he will earn at the end of 2 years?

1) 245

2) 200

3) 260

4) 243

5) 267
Solution:
Scheme A:

The interest is compounded semi-annually.

Interest earned at the end of 2 years





= 10000 1.05
4
10000

10000 1.2155 10000

= 2155

Scheme B:

The rate of interest is 6% per 6 months i.e. 12% p.a.

The interest earned in 2 years = 0.12 2 10000 = 2400

Difference between the interest earned = 2400 2155 = Rs. 245

Hence, option 1.

Group Question


Answer the following questions based on the information given below.


In the year 2004, Deepesh and Devesh together purchased 20 acres of land from
Dinesh. They paid amounts in the ratio 4 : 1. Further Devesh spent Rs. 3 lakhs more
on the land for its development, and thereby they spent equal amounts. They
cultivated mango and orange trees in equal areas. However the ratio of number
of mango trees to that of orange trees was 3 : 2. In the year 2008, the trees yielded
fruits. They sold each mango for Rs. 10. The revenue obtained from the total sale of
mangoes and oranges was 25% of the purchase value of the land. They agreed to
share the realised value equally. The total amounts generated from the sale of
mangoes and oranges were in the ratio of 3 : 2.


46.
3
Marks
What is the ratio of revenue per acre of oranges to that of mangoes?

1) 3 : 2

2) 2 : 3

3) 1 : 1

4) 4 : 9

5) 9 : 4
Solution:
Oranges and mangoes are cultivated on equal areas, the ratio of revenue

per acre of oranges to that of mangoes is the same as the ratio of revenue from
oranges to that of mangoes, which is 2 : 3.

Hence, option 2.


47.
3
Marks
The amount received (in Rs.) by Devesh in the year 2008 is

1) 62500

2) 750000

3) 100000

4) 125000

5) 300000
Solution:
Let Deepesh pay 4x lakhs and let Devesh pay x lakhs for the land. Devesh
spends Rs. 3 lakhs more.

4x = x + 3 i.e. x = 1

Total purchase value of the land = 4x + x = 5 lakhs

Total amount realized from sale of both mangoes and oranges = 25% of 5
lakhs = Rs. 1.25 lakhs

Amount received by Devesh in the year 2008 = 125000/2 = Rs. 62,500

Hence, option 1.


48.
3
Marks
The total number of mangoes produced is

1) 5000

2) 50000

3) 6250

4) 62500

5) 7500
Solution:
From the solution to the previous question we get that the total revenue
generated from the sale of mangoes and oranges = Rs. 1.25 lakhs

Revenue from mangoes = (3/5) 1.25 lakhs = Rs. 75,000

Revenue from oranges = (2/5) 1.25 lakhs = Rs. 50,000

Total revenue from mangoes = Rs. 75,000

Selling price of each mango = Rs. 10

Number of mangoes = 75000/10 = 7500

Hence, option 5.


Section II



1.
3
Answer the following questions based on the information given below.
Marks With or without religion, you would have good people doing good things and evil
people doing evil things. But for good people to do evil things, that takes
religion.

The above argument rests on which of the following assumptions?

1) Religious people do evil things.

2) Good people are incapable of doing evil things.

3) Every evil deed has religion behind it.

4) People are either good or evil.

5) Good and evil can be measured quantitatively.
Solution:
It is not assumed that religious people generally do evil things, as stated in option
1. The given information only states that for good people to do evil, it requires
religion - this cannot be generalized to ALL religious people.
Option 3 is not assumed because evil people without religion can also commit
evil deeds.
Option 4 is not an assumption because although the argument is about good and
evil people, it does not assume that these are the only two classes in which people
can fall.
Option 5 introduces quantitative- no relation to maindata.
Option 2 is an assumption because the paragraph states if good people have to do
evil things, they are forced by religion to do so- hence they are generally
incapable of doing evil (incapable means: lacking capacity, ability, or
qualification for the purpose or end in view).
Hence, the correct answer is option 2.


2.
3
Marks
During the Physical Training session in a school, some girls were made to stand
in a straight line. Asha was the 4
th
girl from the front, while Nisha was the 3
rd
last
girl. The instructor then asked 5 consecutive girls from the center of the line to
move out of the line. This left exactly 4 girls between Asha and Nisha. How
many girls were in the line initially?

1) 14

2) 15

3) 16

4) 17

5) 18
Solution:
Asha was the 4
th
girl from the front which means that there were 3 girls in front
of Asha.

Nisha was the 3
rd
last girl which means that there were 2 girls behind Nisha.

Thus, the number of girls who are always present in the line are 3 + 1 + 1 + 2
i.e. 7.

After the instructor asked 5 consecutive girls to move out, there were 4 girls left
between Asha and Nisha.

i.e. initially there were 9 girls between Asha and Nisha.

Thus, the number of girls initially in the line = 7 + 9 = 16.

Hence, option 3.


3.
3
Marks
Given below is a passage followed by several statements that can be drawn from
the facts stated in the passage. Examine each statement separately in the context
of the passage and decide whether they are implied from the passage.
First, let us do away with the myth that parents teach language to their children.
No one supposes that parents provide explicit grammar lessons, of course, but
many parents (and some child psychologists who should know better) think that
mothers provide children with implicit lessons. These lessons take the form of a
special speech variety called Motherese: intensive sessions of conversational
give-and-take, with repetitive drills and simplified grammar. (Look at the
doggie! See the doggie? Theres a doggie!)
Statements:
I. Some parents mistakenly think that they teach their children language.
II. Motherese may not particularly helpful.
III. Child psychologists are expected to be aware that parents provide children
with only implicit language lessons.


1) Only I is implicit.

2) Only III is implicit.

3) Both I and II are Implicit.

4) Both II and III are Implicit.

5) All are implicit.
Solution:
I is very much implicit in the first sentence, and forms the basis for the whole
argument.
Since the idea of parents teaching their children language is called a myth, we
can infer that even a special variety of speech like Motherese is not particularly
helpful in doing so. So II is implicit.
III directly contradicts the parenthetical observation in the second sentence, so it
cannot be said to be implicit.
Therefore only assumptions I and II are implicit.
Hence, the correct answer is option 3.

Group Question


Answer the following questions based on the information given below.


In Ramraj society, it is known that only three brands of vehicles are used, i.e. Hero
Honda, Yamaha and Bajaj. 42 families use only one brand, 23 families use exactly
two brands and 10 families use all three brands. It is assumed that each family uses
at least one of these brands.


4.
3
Marks
If 8 families stop using Yamaha and start using Bajaj, then what can be the
maximum number of families that use exactly two brands?

1) 26

2) 32

3) 30

4) 31

5) 23
Solution:

Since every family uses atleast one brand, there is no family that does not use
any brand.

42 families use only one brand

a + b + c = 42

23 families use exactly two brands

d + f + e = 23

10 families use all the three brands.

g = 10

The 8 families that stop using the Yamaha brand and start using the Bajaj
brand can be from any one of the three categories : single brand users, two
brand users or all brand users.

Since the maximum number of families that use two brands is required, this
can happen only if all the 8 families are from the same category.

Therefore, there are three cases possible.

Case 1 : All 8 families are single brand users.

In this case, a + b + c is still 42 because the 8 families get reduced from
Yamaha only and get added to Bajaj only.

Thus, there is no impact on the number of families using two brands i.e. its
value still remains 23.

Case 2 : All 8 families are two brand users.

In this case, d + e + f can have a maximum value of 23 because in a best case
scenario, there are originally no families in the Yamaha and Bajaj only
category and the 8 families get reduced from the Hero Honda and Yamaha
onlyand get added to Bajaj and Hero Honda only category. If there were
some families in the Yamaha and Bajaj only category, then after the shift,
the number of families in this category would have become zero while the
number of families in the Bajaj only category would have increase by 8.
Thus, the number of families that use two brands would have reduced by 8 i.e.
become 15.

Thus, in this case, the value becomes either 15 or remains 23.

Case 3 : All 8 families use all three brands.

Here, the value of g definitely decreases by 8 to become 2. This is because the
families in this category now become two brand users i.e. they fall in the
category Bajaj and Hero Honda only.

Hence, the number of families using two brands becomes 23 + 8 = 31.

Hence, the maximum value is 31.

Hence, option 4.


5.
3
Marks
How many families do not use all the three brands?

1) 50

2) 60

3) 54

4) 65

5) None of the above
Solution:
Since every family uses at least one brand, the total number of families that
uses any one of the brands is 42 + 23 + 10 = 75.

The number of families that uses all three brands = 10

Number of families that dont use all three brands = 75 10 = 65

Hence, option 4.


6.
3
Marks
Answer the question based on the information given in the passage.
One reason given by those who are pro-zoos is that zoos have a very important
role to play in breeding endangered animals in captivity. According to pro-zoo
people, if animals were being taken care of in their natural environment, i.e. in
forests and oceans, then there would have been no need for zoos. But so many
animals are now becoming extinct and it is up to the zoos to somehow salvage
the situation.

Which of the following statements further weakens the argument presented in the
passage?

1) Zoos are primarily seen as entertainment centres.

2)
Zoos do not offer guided tours to visitors, therefore there has been no rise
in awareness.

3)
Studies have found that city zoos play an important role in protecting
endangered species from poachers.

4) Zoos have been able to save many species from the brink of extinction.

5)
Studies show that zoos have a very low success rate when it comes to
conserving endangered animals, due to diseases and defects caused due to
inter-breeding.
Solution:
The main idea expressed in the passage is that zoos play a very important role in
conserving endangered species. To weaken this argument we need a statement
that shows that zoos are not doing this, or are somehow incapable of doing this.
Option 1 is ruled out as it is not relevant to the argument in the passage about
conserving endangered species.
Option 2 is ruled out as there is no relation between awareness and wildlife
conservation in this passage.
Option 3 is ruled out as this is just a rephrasing of what has been stated in the
passage; it doesnt weaken the argument.
Option 4 is ruled out as this would not weaken the argument, rather it would
strengthen it.
If the zoos have a very low success rate in conserving animals and inter-breeding
is leading to diseases and defects, then zoos are not playing a role in conserving
endangered species. This certainly weakens the given argument.
Hence, the correct answer is option 5.


7.
3
Marks
Given below are two statements. Analyze them and mark the option that
correctly states their relationship.
A. I wrote pages after pages on the experiences of my travels.
B. The ink-pot fell from the table and the ink in the pen ran dry.


1) A is the cause and B is the effect.

2) B is the cause and A is the effect.

3) Both A and B are effects of a common cause.

4) Both A and B are the causes of a common effect.

5) Cannot be determined.
Solution:
This is an abstract set of statements that connect logically only on the basis of
certain assumptions. On the face of it, statement A looks like the cause to the fact
mentioned in statement B. However, we have to assume that the author was using
to write the very pen that ran out of ink and he did something for the ink-pot to
fall. These two statements cannot be assumed to be related to each other as it
would render the relationship ambiguous.
Hence, the correct answer is option 5.


8.
3
Marks
Read the two statements given below and choose the statement that logically
follows from the first two.
A. All pygmies are brown.
B. All pygmies are black.


1) Brown is same as black.

2) Some brown is black.

3) Some black are not brown.

4) Some brown are pygmies.

5) Every brown is black.
Solution:

The two given statements can be represented in the form of a Venn diagram.

Option 1 is incorrect because brown and black may not be the same set.

Option 2 is correct because there has to be some brown that are common to black
(the set of all pygmies).

Option 3 is incorrect because brown and black could be the same set.

Option 4 is derived only from statement A and is hence, incorrect.

Option 5 is incorrect as seen in the Venn Diagram.

Hence, option 2.


9.
3
Marks
The statement given below is followed by two statements. Analyze all of them
and determine the correct combination of a course of action.
Illegal hunting of tigers has caused the tiger population of India to reach alarming
levels.
A. Environmental groups should spread awareness about this fact.
B. The government should enforce stricter measures towards upholding the
existing tiger poaching laws.


1) A is the best course of action.

2) B is the best course of action.

3) A followed by B is the best course of action.

4) Either A or B is the best course of action.

5) Neither A nor B are relevant courses of action.
Solution:
Both of these statements seem useful as courses of action when it comes to tiger
poaching. However, this situation is very specific and so generic actions are not
relevant in this case.
Statement A will definitely result in people being more aware of dropping tiger
populations and start caring more but such awareness among the general
population may or may not actually reduce the number of tigers being hunted
illegally. Statement B, on the other hand, targets the crux of the problem, that is
upholding laws more strictly so that the people who hunt these tigers are caught
and stopped from hunting any more tigers. Between these two, statement B is
more relevant as a course of action than statement A.
Hence, the correct answer is option 2.


10.
3
Marks
In each of the following questions, there is a statement followed by a set of
arguments. You are expected to classify the argument as Strong or Weak.
Strong arguments are important and directly related to the question. Weak
arguments may not be directly related or may be related to trivial aspects of the
question and may be of lesser importance.
Choose the best option as per this classification.
Should there be a statutory limit on the amount spent on weddings, in view of the
display of wealth witnessed at the weddings of politicians and industrialists?
I. Yes. Conspicuous consumption by public authorities undermines efforts
to cultivate healthy economic habits across all classes, which is important
for the success of the nation.
II. No. A large wedding creates jobs for people in several industries, from
florists to caterers to photographers to bridal fairs. It boosts the local
economy.


1) Only I is strong.

2) Only II is strong.

3) Both I and II are strong.

4) Either I or II is strong.

5) Neither I nor II is strong.
Solution:
Argument I is not supported by strong facts. Firstly, in a democracy, one cannot
lay a limit on the personal spending behaviour of a person. Secondly, across all
classes is incorrect. People from the middle and lower economic classes will not
have lavish spending habits, therefore, do not need to take an effort to cultivate
healthy economic habits with respect to lavish weddings. Hence, argument I is
weak.
Argument II is supported with unimpeachable facts. Therefore, argument II is
strong.
Hence, the correct answer is option 2.

Group Question


Answer the following questions based on the information given below.


A clerk has been given the task of arranging a round table conference that will be
attended by 8 delegates from different countries : A, B, C, D, E, F, G and H.

The main job of the clerk is to assign the name of the delegate to the chair meant for
that delegate.

The clerk has been given the following information
i. The chairs are numbered from 1 to 8.
ii. The delegate from country A will sit opposite to the delegate from country E.
iii. The delegate from country C sits adjacent to the delegate from country D.
iv. The delegate from country F sits opposite to the delegate from country G.
v. The delegate from country B sits on the right hand side of the delegate from
country A.
vi. The delegate from country H sits adjacent to the delegate from country E.
The clerk has been further informed that the delegate from country A sits on the seat
numbered 1 and that the seats have been arranged in clockwise fashion.

Try helping the clerk in this job by answering the following.


11.
3
Marks
Whose name will the clerk write on seat number 2?

1) Delegate from G

2) Delegate from F

3) Delegate from D

4) Delegate from C

5) Cannot be determined
Solution:
Since it is given that the delegate from country A sits on seat number 1 and
that the seats are numbered in clockwise order, fix A's position and number
the other seats accordingly.

Now, the delegate from country E sits exactly opposite to the delegate from
country A.

Thus, the delegate from country E sits on seat number 5.

Since the delegate from country B sits on the right hand side of the delegate
from A, the delegate from B can sit on any one seat from seat numbers 6, 7
and 8.

Now, the delegates from countries C and D sit adjacent to each other and the
delegates from countries G and F are exactly opposite each other.

This implies that the delegates from countries C, D and one out of countries G
and F have to occupy three adjacent seats.

Hence, the three delegates mentioned above have to occupy seat numbers 2, 3
and 4 (in no particular order).

Now, the delegate from country H sits adjacent to the delegate from country
E.

This means that the delegate from country H can sit on one of seats 4 and 6.

But, as seen above, seat 4 will be occupied by one delegate from C, D and
G/F.

Hence, the delegate from country H will be on seat number 6.

Consequently, the delegate from country G or F cannot be opposite to seat
number 6 i.e. on seat number 2.

Similarly, this delegate cannot be on seat number 3 because the delegates
from C and D have to be together.

Hence, one delegate from country G and F will be on seat number 4 while the
other will be opposite him i.e. on seat number 8.

Consequently, the delegates from countries C and D will be on seat numbers 2
and 3 (in no particular order).

Hence, the delegate from country B can only be in seat number 7.

Thus, the final arrangement is as shown below.



Therefore, it is clear from the figure that seat number 2 will be occupied by
either C or D.

Hence, option 5.


12.
3 Marks
Who will sit on the seat number 7?

1) Delegate from G

2) Delegate from H

3) Delegate from F

4) Delegate from B

5) Cannot be determined
Solution:
Consider the final arrangement obtained in the solution to the first question.

It is evident that the delegate from country B sits on the seat numbered 7.

Hence, option 4.


13.
3 Marks
On which seat number will H be seated?

1) 4

2) 6

3) 7

4) 2

5) Cannot be determined
Solution:
Consider the final arrangement obtained in the solution to the first question.

You can observe that H sits on seat number 6.

Hence, option 2.


14.
3
Marks
Read the two statements given below and choose the statement that logically
follows from the first two.
A. All Foreigners are Stupid.
B. All Stupid are Awkward.


1) Some Awkward are Stupid.

2) Some Foreigners may be Stupid but not Awkward.

3) Some Stupid are Foreigners.

4) All Awkward are Foreigners.

5) Some Awkward are Foreigners.
Solution:

The two given statements can be represented in the form of a Venn diagram as
shown.

Options 1 and 3 are incorrect as they are derived from one of the given
statements only.

Option 2 is incorrect as such a case is not possible (as seen in the Venn
Diagram).

Option 4 is not correct. Though it may be one of the possible cases, it is not
necessarily true.

Option 5 is correct as can be seen from the diagram.

Hence, option 5.


15.
3
Marks
A coding machine generates a code for words in a certain manner such
that the code for STRANGER is 19-T-18-A-14-G-5-R. What will be the code
generated by this machine for the word IMPORTANT?

1) 10-M-16-O-18-T-1-N-20

2) 9-M-16-O-18-T-1-N-21

3) 9-M-14-O-18-T-1-N-20

4) 9-M-16-O-18-T-1-N-20

5) 4-I-1-L-16-G-21-E
Solution:
Looking at the code for STRANGER, it is clear that starting from the first letter,
every alternate letter is replaced by its position in the alphabet. The other set of
alternate letters stay as they are.

Word: S T R A N G E R

Position: 19 na 18 na 14 na 5 na

Code: 19 T 18 A 14 G 5 R

Now, consider the word IMPORTANT.

Word: I M P O R T A N
T

Position: 9 na 16 na 18 na 1 na 20

Code: 9 M 16 O 18 T 1 N 20

The code for IMPORTANT is 9-M-16-O-18-T-1-N-20.

Hence, option 4.


16.
3
Marks
Read each of the passages and answer the questions that follow.
Todays society is reeling from the effects of constant advertising messages.
These advertisements promote harmful and dangerous things that weaken social
fabric and lead to the ill-health of young people. Advertisers promote unhealthy
things and are not genuinely concerned about society. Fast food adverts are a
major reason why children today are overweight.

Which of the following statements, if true, best weakens the argument presented
in the passage?

1) Advertising affecting obesity is just a myth.

2) Society cannot deal with the pressure of advertising.

3) Fast food is the leading cause of obesity.

4)
Researchers have found that most young people tend to be quite lean till
the age of 20.

5)
Advertisements that promote unhealthy products are not as common as
believed and fast food companies are now promoting their healthier dishes
on offer.
Solution:
The main idea presented in the passage is that advertisements promote dangerous
things that have a negative impact on people. The example is that of fast food
advertisements targeted at young people. To weaken the argument we need a
statement that shows that all advertisements do not promote unhealthy things and
that fast food companies do not promote fattening food products which leads to
children being overweight.
Option 1 does not weaken the argument as there is no concrete data.
Option 2 simply rephrases what has already been stated in the passage.
Option 3 supports the passage.
Option 4 is unrelated to the effects of advertising and merely adds more data to
the passage.
Option 5 is the correct answer since it weakens the argument by stating that
advertisements that promote unhealthy products are rare (this goes against the
statement that, Todays society is reeling from the effects of constant advertising
messages). Secondly, it also says that fast-food companies have been promoting
their healthier dishes, so the advertisements promoting these kinds of food cannot
be held accountable for obesity among the young.
Hence, the correct answer is option 5.

Group Question


Answer the following questions based on the information given below.


Ten friends go to watch the IPL match. The seating arrangement is such that 5 people
sit in one row while the other 5 sit in the other row. The following information is
known about them.
1. The rows are one behind another.
2. There are only 4 girls in the group - Shalini, Priya, Priyanka and Sonia.
3. Each row has an equal number of girls.
4. Shalini does not sit in the same row as Ram.
5. Priya and Priyanka sit in different rows.
6. Ramesh and Suresh have to sit together if Karan and Sonia sit together and
vice versa.
7. Karan sits in the 1
st
row.
8. If Abeer sits in the 1
st
row, then Prem sits behind Suresh otherwise
Suresh sits behind Prem.
9. Shalini always sits to the left of Ramesh.



17.
3
Marks
If Ram sits in the 1
st
row, who can sit to his left from the following?

1) Suresh

2) Abeer

3) Ramesh

4) Prem

5) Cannot be determined
Solution:
If Ram is sitting in the 1
st
row, Shalini has to sit in the 2
nd
row.

Shalini always sits to the left of Ramesh and hence Ramesh is also sitting in
the 2
nd
row.

Now, one of Priya or Priyanka must sit in the 2
nd
row.

Now, if Karan and Sonia sit separately, then Sonia has to sit in the 2
nd
row (as
Karan always sits in the 1
st
row).

However, this is not possible as there are already 2 girls in the 2
nd
row.

Thus, Karan and Sonia sit together in 1
st
row which means Suresh has to
sit with Ramesh in the 2
nd
row.

Thus, Prem has to sit in the 1
st
row which means Abeer is in the 2
nd
row.

Of the given options, only Prem is seated in the 1
st
row.

Hence, option 4.


18.
3
Marks
If Sonia sits in the 2
nd
row, who will definitely be seated in the 1
st
row?

1) Ramesh

2) Suresh

3) Abeer

4) Priya

5) Cannot be determined
Solution:
Now, one of Priya or Priyanka would be seated with Sonia in the 2
nd
row.

Shalini has to sit in the 1
st
row which means Ramesh would also sit in the
1
st
row.

Also, Ram has to sit in the 2
nd
row.

Since Karan always sits in the 1
st
row and Sonia is now in the 2
nd
row, they
are sitting separately.

Ramesh and Suresh should be sitting in different rows i.e. Suresh has to sit
in the 2
nd
row.

Now, if Abeer sits in the 1
st
row, Prem has to sit behind Suresh.

But, since Suresh is in the 2
nd
row, Prem has to sit in the 1
st
row which means
Abeer is in the 2
nd
row.

Hence, option 1.


19.
3
Marks
Boys and girls are alternately seated in the 1
st
row. If Sonia sits in the 1
st

row and Suresh is sitting at the right most end of one of the rows, who will
occupy the 3
rd
place in the 1
st
row?

1) Prem

2) Abeer

3) Karan

4) Ram

5) Cannot be determined
Solution:
If Sonia sits in 1
st
row, she and Karan are sitting together.

Ramesh and Suresh are also sitting together.

Let us assume that Suresh is in the right most end of the 1
st
row.

Now, one of Priya or Priyanka have to sit in the 1
st
row.

Since all the five seats in the 1
st
row are occupied, Shalini will have to sit in
the 2
nd
row with Ram, which contradicts the given data.

Suresh is sitting in the right most end of the 2
nd
row.

Now, Prem must sit ahead of Suresh and hence he occupies the right most end
of the 1
st
row.

i.e. Abeer is in the 2
nd
row.

Ramesh will sit in the 2
nd
row with Suresh and Shalini will be to the left of
Ramesh and the remaining vacant spot will be occupied by one of
Priya/Priyanka.

People sitting in the 1
st
row: Karan, Sonia, Ram, Priya/Priyanka, Prem.

We only know Prem's position and that girls and boys are sitting alternately.

One of Karan or Ram can sit in the 3
rd
place.

Hence, option 5.


20.
3
Marks
If CHIKU = 20 and BRINJAL = 42, what is the value of PINEAPPLE?

1) 50

2) 72

3) 44

4) 69

5) 41
Solution:
The logic here is that the number of letters in the word is multiplied by 1 less
than the count.

The word CHIKU has 5 letters.

It is coded as 5 4 = 20.

Similarly, the word BRINJAL has 7 letters, hence coded as 7 6 = 42.

Using the same logic, PINEAPPLE is coded as 9 8 = 72.

Hence, option 2.


21.
3
Marks
Four of the five options are equal in magnitude. Which of the options has a
magnitude different from the others?
1. 5465 4287
2. 1178 + 2145 1015 1130
3. (1100 2) + (1884 3)
4. 9 99 + 6 66 3 33
5. 666 + 333 + 111 + 66 + 2


1) 1

2) 2

3) 3

4) 4

5) 5
Solution:
1. 5465 4287 = 1178
2. 1178 + 2145 1015 1130 = 1178 + 2145 2145 = 1178
3. (1100 2) + (1884 3) = 550 + 628 = 1178
4. 9 99 + 6 66 3 33 = 891 + 396 99 = 1287 99 = 1188
5. 666 + 333 + 111 + 66 + 2 = 1178
Hence, option 4.

Group Question


Answer the following questions based on the information given below.


Four friends Ajay, Bob, Chetan and Dan live in house numbers 1, 2, 3 and 4,
though not necessarily in the same order. They own the following brand of cars :
Skoda, Sumo, Santro and Swift among them. No two persons own the same brand of
car. The following information is known about them.
A. Neither Ajay nor Dan live in an even numbered house.
B. The person in house 2 drives a Skoda.
C. The owner of the Swift is in house number 4.
D. Ajay does not own a Sumo.
E. Bob drives the Swift.



22.
3
Marks
In what house number does Ajay live?

1) 1

2) 3

3) 1 or 3

4) 2

5) 2 or 4
Solution:
There are 3 parameters here : Person, Brand of Car and House number.

Now, Bob drives the Swift and the owner of the Swift stays in house number
4.

Therefore, Bob lives in house number 4.

Bob - Swift - House 4.

Neither Ajay nor Dan stay in even numbered houses.

Therefore, these two stay in house numbers 1 and 3 (in no specific order).

Hence, Chetan has to stay in house no 2.

Since the person in house number 2 drives a Skoda, Chetan drives a Skoda.

Chetan - Skoda - House 2.

Now, Ajay can drive a Sumo or a Santro.

However, it is given that Ajay does not drive a Sumo.

Therefore, Ajay drives a Santro and Dan drives a Sumo.

However, it cannot be determined whether Ajay stays in house number 1 or
3. He can stay in any one of the two.

Hence, option 3.


23.
3
Marks
Which car does Dan drive?

1) Santro

2) Sumo

3) Swift

4) Skoda

5) Cannot be determined
Solution:
From the solution to the first question of this set, it is obvious that Dan drives
the Sumo.

Hence, option 2.


24.
3
Marks
If Ajay lives in house 1, then in which house does Dan live?

1) 2

2) 3

3) 4

4) 3 or 4

5) None of these
Solution:
From the solution to the first question of this set, Ajay and Dan stay in house
numbers 1 and 3 (in no specific order).

Therefore, if Ajay stays in house number 1, Dan has to stay in house number
3.

Hence, option 2.


25.
3
Marks
On a particular evening, Mansi was heading home from office, walking towards
the north. She crossed her daughter Latika mid way on the route. Mansi observed
that her shadow was falling towards Latikas right. In which direction was Latika
moving?

1) North

2) South

3) East

4) West

5) Indeterminate
Solution:
In the evening, since the sun is in the west, the shadow of an object will fall
towards the east.

Since Mansi was walking towards the north, her shadow fell on her right hand
side.

For this shadow to fall on Latika's right, there can be two possiblities:

Case 1 : Latika was walking towards the north.

If Latika was also walking towards the north and was either on the left of Mansi
or parallel to Mansi (on the right side) such that the distance between Latika and
Mansi (at the point of crossing) was less than the length of the shadow, Mansi's
shadow would fall on Latika's right.

Hence, the given condition would be satisfied.

Case 2 : Latika was walking towards the south.

If Latika was walking towards the south and was parallel to Mansi (on the right
side) such that the distance between Latika and Mansi (at the point of crossing)
was more than the length of the shadow, Mansi's shadow would fall on Latika's
right.

Hence, the given condition would still be satisfied.

Hence, Latika could have been walking towards either the north or the south.

Hence, option 5.

Group Question


Answer the following questions based on the information given below.


The columns and rows of matrix 1 are numbered 0 to 4 and that of matrix 2 from 5 to
9. A letter from these matrices is represented first by its row and next by column
number e.g. O can be represented as 23, 34 and so on. In each of the questions that
follow, find the set of codes that represents the word in that corresponding question.




26.
3
Marks
MORAL

1) 43, 11, 77, 02, 96

2) 32, 33, 75, 44, 85

3) 22, 23, 66, 21, 65

4) 01, 23, 55, 03, 77

5) 14, 23, 55, 33, 69
Solution:
M can be coded as 01, 14, 22, 32
and 43

O can be coded as 00, 11, 23, 34
and 40.

Hence, option 2 can be eliminated.

R can be coded as 55, 66, 75, 88
and 99.

Hence, option 1 can be eliminated.

A can be coded as 02, 10, 21, 33
and 44.

Hence, option 4 can be eliminated.

L can be coded as 56, 69, 77, 85
and 96.

Hence, option 3 can be eliminated.

Only option 5 contains all the
correct codes.

Hence, option 5.


27.
3 Marks
TONE

1) 31, 23, 59, 68

2) 04, 11, 76, 59

3) 20, 32, 67, 89

4) 31, 00, 87, 57

5) 31, 43, 59, 65
Solution:
T can be coded as 04, 12, 20, 31 and 42.

O can be coded as 00, 11, 23, 34 and 40.

Hence, options 3 and 5 can be eliminated.

N can be coded as 59, 67, 78, 86 and 95.

Hence, options 2 and 4 can be eliminated.

E can be coded as 57, 68, 76, 89 and 98.

Only option 1 contains all the correct codes.

Hence, option 1.


28.
3 Marks
KITE

1) 87, 31, 04, 76

2) 65, 13, 24, 89

3) 97, 24, 02, 57

4) 65, 03, 12, 98

5) 79, 30, 98, 12
Solution:
K can be coded as 58, 65, 79, 87 and 97.

I can be coded as 03, 13, 24, 30 and 41.

Hence, option 1 can be eliminated.

T can be coded as 04, 12, 20, 31 and 42.

Hence, options 2, 3 and 5 can be eliminated.

Only option 4 contains all the correct codes.

Hence, option 4.


29.
3
In a certain code language, if the word MAGIC is coded as 16-24-10-6-6, and the
Marks word BREAD is coded as 5-15-8-24-7, then how is the word RITUAL coded in
that language?

1) 21-6-23-18-5-9

2) 21-6-25-18-4-9

3) 21-7-23-18-4-9

4) 22-6-23-18-4-9

5) 21-6-23-18-4-9
Solution:
Consider the alphabetical position of each letter of the word MAGIC.

Word: M A G I C

Position: 13 1 7 9 3

Logic: +3 3 +3 3 +3

Code: 16 24 10 6 6

Similarly, consider the alphabetical position of each letter of the word BREAD

Word: B R E A D

Position: 2 18 5 1 4

Logic: +3 3 +3 3 +3

Code: 5 15 8 24 7

Thus, starting from the letter at the extreme left, each letter is alternately replaced
by a letter 3 places after it (or before it) in the alphabet.

Similarly, consider the position of each letter of the word RITUAL.

Word: R I T U A L

Position: 18 9 20 21 1 12

Logic: +3 3 +3 3 +3 3

Code: 21 6 23 18 4 9

So, the code for the word RITUAL is 21-6-23-18-4-9.

Hence, option 5.


30.
3
Marks
If the representation (symbols) of Addition and Subtraction are interchanged and
representation (symbols) of Multiplication and Division are interchanged, then
what is the value of the following expression (L M)/(X + M) (M + I) (L
I)?

The values of the Variables are:

I = 1

L = 10

M = 100

V = 500

X = 1000

1) 11000

2) 99109

3) 110001

4) 99009

5) None of these
Solution:
The expression can be decoded as,

(L M)/(X + M) (M + I) (L I) = (L + M) (X M) + (M I)/(L + I)

= (10 + 100) (1000 100) + (100 1)/(10 + 1)

= 99009

Hence, option 4.


31.
3
Marks
Read the two statements given below and choose the statement that logically
follows from the first two.
A. All laborers are workers.
B. Some workers are builders.


1) Some laborers are builders.

2) All laborers are builders.

3) Some builders are workers.

4) All laborers may be builders.

5) Some laborers are not builders.
Solution:

A possibility arising from the two given statements can be represented in the form
of a Venn diagram as shown.

Options 1 and 2 are incorrect as the workers who are laborers may not be
builders.

Option 3 is incorrect as it logically follows only from statement B.

Option 4 is correct because of the word may which means that at least in one of
the cases, all laborers are builders.

Option 5 is incorrect because there can be a case where all laborers are builders.

Hence, option 4.


32.
3
Marks
Given below is a passage followed by several statements that can be drawn from
the facts stated in the passage. Examine each statement separately in the context
of the passage and decide whether they are implied from the passage.
Not so long ago, Chongqing hardly seemed destined for greatness. Quite the
contrary. It was by no means inevitable that a misty, ancient town built on the
bluffs of the Yangtze would become the fastest-sprawling global metropolis.
Indeed, the citys meteoric rise is difficult to explain by logic alone. But it is
emblematic: Timing, luck, geography and government largesse all played a part.
Statements:
I. Surprisingly enough, Chongqing is not a great city now.
II. Chongqing became great very fast.
III. Chongqings government was not a factor in its success.


1) Only I is implicit.

2) Only II is implicit.

3) Both I and III are Implicit.

4) Both II and III are implicit.

5) Both I and II are implicit.
Solution:
Statement II is implied from meteoric rise, destined for greatness and fastest-
sprawling global metropolis
Statements I and III contradict the data given in the passage.
Hence, the correct answer is option 2.


33.
3
Marks
The result of class 10 is declared. Half of those who failed in English managed to
pass in Mathematics and 1/3
rd
of who failed in Mathematics managed to pass in
English. Every student of this class passed in exactly one subject. The ratio of the
number of students who passed in English to the number of students who passed
in Mathematics is,

1) 1 : 3

2) 2 : 3

3) 1 : 2

4) 2 : 1

5) 3 : 2
Solution:
Let x be the number of students that passed only in Mathematics, y be the number
of students that passed only in English and z be the number of students that failed
in both exams.

Since every student of the class passed in exactly one subject, the number of
students that passed in English = the number of students that passed in English
only. The same logic applies for Mathematics.

Then, the number of students that failed in Mathematics is y + z

y + z = 3y

z = 2y

Similarly,

The number of students who failed in English is x + z

x + z = 2x

x = z

y : x = 1 : 2

Hence, option 3.

Group Question


Answer the following questions based on the information given below.


Among 8 persons A, B, C, D, E, F, G and H, there are three married couples and two
pairs of siblings. It is known that:
i. Ds father-in-law is E and Fs daughter-in-laws brother is G.
ii. Members in each pair of siblings are of different genders.
iii. As mother H likes B, the daughter of C and E.
iv. A is Bs sister-in-law and two of the three married couples have two children
each.



34.
3
Marks
Who all belong to the same generation?

1) F, A, G, E

2) F, H, C, G

3) A, H, B, C

4) A, D, C, G

5) A, D, B, G
Solution:
It is given that there are three couples, two pairs of siblings and two out of
three couples have two children.

If a rectangle represents males, a circle represents females and a double line
represents a couple, the basic family tree would be:



From conditions

(i) Ds father-in-law is E and Fs daughter-in-laws brother is G.

(iii) B is the daughter of C and E.

We know that D and B are the children of different couples.

Also from (iv) and (i) A is a female and G is a male.

A, D, G and B belong to the same generation.

Also, as B is a female; B and G form one pair of siblings and A and D form
the other.

B and D form a married couple.

We know C and E form a pair of married couple where E is a male.

H and F are the other married couple.

We know that H is female,

F is male.

Thus the family tree obtained is:



A, D, B and G belong to the younger generation.

Hence, option 5.


35.
3 Marks
Who are the males among the eight?

1) F, G, D, E

2) F, A, G, E

3) F, G, A, B

4) E, D, G, A

5) G, C, D, B
Solution:
From the figure, we can see that F, G, D and E are males.

Hence, option 1.


36.
3 Marks
How is D related to B?

1) Brother

2) Wife

3) Husband

4) Sister

5) Father-in-law
Solution:
As it can be seen, D is Bs husband.

Hence, option 3.

Group Question


Answer the following questions based on the information given below.


Shreya has with her a cube made up of 27 smaller cubes. She names each face of the
larger cube with an alphabet from AF. She then starts writing a number from 154
on each visible face of the smaller cube such that the face A gets the lowest sum
followed by face B and so on. Shreya writes the digits in such a way that no two
consecutive integers are written on a particular face.


37.
3
What can be the least total on the face D?

1) 180
Marks

2) 162

3) 252

4) 168

5) None of these
Solution:
Face A < Face B < Face C < Face D

Since there are 4 faces under consideration, consider the numbers to be in
groups of 4 i.e. group size = number of faces under consideration.

To make the inequality possible and to ensure the least total for face D, assign
the first four natural numbers to faces A, B, C and D respectively.

Now, if the same process is followed i.e. the number 5 is assigned to face A,
the number on face D will be 8 i.e. the largest of this group of 4. This will not
help minimise the sum. Also, if one assigns the largest number of a group of 4
numbers to the face where the smallest number was assigned in the previous
iteration, the gap between the sum on the faces will not grow.

Therefore, assign 5 to face B, 6 to face C, 7 to face D and 8 to face A.

In general, follow the algorithm given below:

1) Assign the first four natural numbers to A, B, C and D respectively.

2) For the next four, assign the largest number of that group to the face that
had the smallest number of the previous group.

3) Since each face will have 9 small cubes, follow this till 9n natural numbers
are not exhausted. Here, n is the number of faces under consideration (in this
case, 4).

Thus, the numbers on the first four faces can be shown as follows:



Thus, the least possible total for the face D is 168.

Hence, option 4.


38.
3
Marks
What is the least total on the face F?

1) 255

2) 258

3) 260

4) 262

5) None of these
Solution:
Using the logic given in the problem above, the numbers in the 6 cubes can
be found as shown below.





Thus, the least total on face F is 255

Hence, option 1.


39.
3
Marks
What is the difference between the total on face A and face B when the total
on face F is the least?

1) 1

2) 2

3) 3

4) 4

5) 5
Solution:
Consider the arrangement of numbers obtained in the previous solution.

In this arrangement, the total on face F is the least.

In this case, the difference between the sum on face A and face B is 3.

Hence, option 3.


40.
3
What is the maximum total for the face C?

1) 325

Marks

2) 327

3) 329

4) 331

5) None of these
Solution:
For the total on face C to be the highest, try and minimise the total on faces A
and B.

Each face has 9 numbers on it.

To minimise the total on faces A and B, assign the first 18 natural numbers
alternately to faces A and B respectively.

Now, to maximise the total on face C, follow the same procedure as outlined
in the first two questions staring from the number 19 and ending at 54.

Hence, the final arrangement is as shown below.





As per this arrangement, the maximum total on face C is 327.

Hence, option 2.


Section III



1.
3
Marks
Choose the word which is OPPOSITE in meaning to the word given in
CAPITAL LETTERS:
AZOIC

1) Animate

2) Sterile

3) Massive

4) Persistent

5) Azure
Solution:
Azoic means inanimate; without life. It is believed to be of an age that
preceded the first appearance of life. It is characterised by having left no evidence
of life in the form of organic remains.
Animate means living; possessing life.
Sterile means barren.
Massive means huge.
Persistent means determined to continue despite obstacles.
Azure means shades of Blue.
Hence, the correct answer is option 1.


2.
3
Marks
Each of the questions below contains a paragraph followed by alternative
summaries. Choose the option that best captures the essence of the paragraph.
Cannibalism has been occasionally practiced as a last resort by people suffering
from famine. In the US, the group of settlers known as the Donner party resorted
to cannibalism while snowbound in the mountains for the winter. The last
survivors of Sir John Franklin's Expedition were found to have resorted to
cannibalism in their final push across King William Island towards the Back
River. There are disputed claims that cannibalism was widespread during the
famine of Ukraine in the 1930s, during the Siege of Leningrad in World War II,
and during the Chinese Civil War and the Great Leap Forward in the People's
Republic of China. There were also rumors of several cannibalism outbreaks
during World War II in the Nazi concentration camps where the prisoners were
malnourished. Cannibalism was also practiced by Japanese troops as recently as
World War II in the Pacific theater. A more recent example is of leaked stories
from North Korean refugees of cannibalism practiced during and after a famine
that occurred sometime between 1995 and 1997.

1)
History claims that Cannibalism, though uncommon, has been practiced for
a very long time. Recent examples also show that cannibalism is not
restricted to a region or country but has been practiced, at some point in
time, in various parts of the world.

2)
There were rumors of several cannibalism outbreaks during World War II
in the Nazi concentration camps where the prisoners were malnourished.
Cannibalism was also practiced by Japanese troops as recently as World
War II in the Pacific theater.

3)
The stories of cannibalism outbreak during World War II and Ukraine
famine strengthen the claims of similar outbreaks elsewhere in history.
Some of the claims, though disputed, seem to be true based on such
examples. Malnutrition, not religion, is believed to be the major reason for
such a practice.

4)
Cannibalism is a rare practice which has been followed in some parts of the
world at times of crisis during wars and famines, which lead to people
taking such drastic measures. Traces of this practice have been found
around the globe, in China, USA, Germany, Japan and many others.

5)
Cannibalism is only practiced at times of war and famines when there is a
lack of food. This practice has been reported in several parts of the world. It
is uncommon, but is said to have broken out in USA, Germany, China,
Japan and many others.
Solution:
The passage does not introduce or describe the practice of cannibalism. It only
mentions it as being a last resort and goes on to give examples of the places where
it has been reported to have broken out.
Option 1, though close, does not cover all the points properly. It also does not
mention any of the examples from the passage.
Option 2 simply reproduces parts of the passage without summarizing it.
Option 3 introduces external data- religion.
Option 5 states that it is only practiced at times of war and famines. The passage
does not mention the exclusivity factor.
Option 4 is correct as it covers all the points exactly the way they are mentioned
in the passage without altering any piece of information. It also touches upon the
examples briefly.
Hence, the correct answer is option 4.


3.
3
Marks
Fill in the blanks in the passage with the most appropriate set of words from the
options for each blank.
Although the principle that events ____ the heavens are mirrored ____ those on
Earth were once generally held ____ most traditions ____ astrology around the
world, ____ the West there has historically been a debate among astrologers ____
the nature of the mechanism behind astrology.

1) in, by, in, of, in, over

2) of, by, by, in, in, over

3) in, in, in, in, inside, over

4) in, by, in, in, in, under

5) of, by, in, of, in, over
Solution:
Options 2 and 5 are incorrect as we need a preposition to describe the events that
occur in the heavens. Hence of is an incorrect preposition for the first blank.
Further, in signifies within the limits or boundary of. The correct preposition
for the fourth blank will be of. Options 3 and 4 can be eliminated.
Hence, the correct answer is option 1.

Group Question


A passage is followed by questions pertaining to the passage. Read the passage and
answer the questions. Choose the most appropriate answer.


Chromosome linkage studies of sexual orientation have indicated the presence of
multiple contributing genetic factors throughout the genome. In 1993, Dean Hamer
and colleagues published findings from a linkage analysis of a sample of 76 gay
brothers and their families. Hamer found that the gay men had more gay male uncles
and cousins on the maternal side of the family than on the paternal side. Gay brothers
who showed this maternal pedigree were then tested for X chromosome linkage,
using twenty-two markers on the X chromosome to test for similar alleles. In another
finding, thirty-three of the forty sibling pairs tested were found to have similar alleles
in the distal region of Xq28, which was significantly higher than the expected rates of
50% for fraternal brothers. This was popularly (but inaccurately) dubbed as the 'gay
gene' in the media, causing significant controversy.

A later analysis by Hu replicated and refined these findings. This study revealed that
67% of gay brothers in a new saturated sample shared a marker on the X chromosome
at Xq28. Sanders replicated the study, finding 66% Xq28 marker sharing in 54 pairs
of gay brothers. Although two other studies failed to find a preponderance of gay
relatives in the maternal line of homosexual men, a rigorous replication of the
maternal loading was reported on samples in Italy in England. One study by Rice
failed to replicate the Xq28 linkage results. Meta-analysis of all available linkage data
indicates a significant link to Xq28, but also indicates that additional genes must be
present to account for the full heritability of sexual orientation.

Mustanski performed a full-genome scan (instead of just an X chromosome scan) on
individuals and families previously reported on in Hamer and Hu, as well as
additional new subjects. With the larger sample set and complete genome scan, the
study found somewhat reduced linkage for Xq28 than reported by Hamer et al.
However, they did find other markers with significant likelihood scores at 8p12, 7q36
and 10q26. Interestingly, one of the links showed highly significant maternal loading,
thus further confirming the previous family studies.

A recent study suggests linkage between a mother's genetic make-up and
homosexuality of her sons. Women have two X chromosomes, one of which is
"switched off". The inactivation of the X chromosome occurs randomly throughout
the embryo, resulting in cells that are mosaic with respect to which chromosome is
active. In some cases though, it appears that this switching off can occur in a non-
random fashion. Bocklandt reported that, in mothers of homosexual men, the number
of women with extreme skewing of X chromosome inactivation is significantly
higher than in mothers without gay sons. 13% of mothers with one gay son, and 23%
of mothers with two gay sons showed extreme skewing, compared to 4% percent of
mothers without gay sons.

Blanchard and Klassen reported that each older brother increases the odds of a man
being gay by 33%. This is now "one of the most reliable epidemiological variables
ever identified in the study of sexual orientation." To explain this finding, it has been
proposed that male fetuses provoke a maternal immune reaction that becomes
stronger with each successive male fetus. Male fetuses produce HY antigens which
are "almost certainly involved in the sexual differentiation of vertebrates." It is this
antigen which maternal H-Y antibodies are proposed to both react to and 'remember'.
Successive male fetuses are then attacked by H-Y antibodies which somehow
decrease the ability of H-Y antigens to perform their usual function in brain
masculinisation.


4.
3
Marks
Which of these factors may not lead to a gay offspring?

1) A gay maternal uncle

2) A marker on the X chromosome at Xq28

3) Extreme skewing of X chromosome activation

4) Three older brothers

5) Fewer H-Y antigens
Solution:
According to the first paragraph, a gay maternal uncle does increase the
chances of a gay offspring. Therefore, option 1 is incorrect.
According to the second paragraph, This study revealed that 67% of gay
brothers ... shared a marker on the X chromosome at Xq28. Therefore, a
marker on the X chromosome at Xq28 may be found in gay offspring, making
option 2 incorrect.
Option 4 is incorrect as the last paragraph states, Blanchard and Klassen
reported that each older brother increases the odds of a man being gay by
33%.
According to the last paragraph, Male fetuses produce HY antigens which are
"almost certainly involved in the sexual differentiation of vertebrates." ...
Successive male fetuses are then attacked by H-Y antibodies which somehow
decrease the ability of H-Y antigens to perform their usual function in brain
masculinisation. Thus fewer H-Y antigens will result in a decreased brain
masculinisation function, making it more likely to lead to homosexuality.
Therefore, option 5 is incorrect.
According to the fourth paragraph, Bocklandt reported that, in mothers of
homosexual men, the number of women with extreme skewing of X
chromosome inactivation is significantly higher than in mothers without gay
sons. Thus, it is the extreme skewing of X chromosome INACTIVATION,
and not activation, that may lead to a gay offspring.
Hence, the correct answer is option 3.


5.
3
Marks
Interestingly, one of the links showed highly significant maternal loading,
thus further confirming the previous family studies. Why does the author
begin this statement with the word interestingly?

1)
Because though the studys focus was to use a full genome scan to
determine the veracity of the gay gene and to see if any other genes
affected homosexuality, it also ended up gathering evidence in support
of the much debated topic of maternal loading.

2)
Because it is interesting that maternal loading is still debated when the
evidence clearly states otherwise; a full genome scan reveals that genetic
factors are responsible for homosexuality.

3)
Genetic markers like the ones on 8p12, 7q36 and 10q26 prove that
homosexuality is a genetic defect, as revealed by a full genome scan,
and that maternal loading has very little to do with it.

4)
It is interesting to note that maternal loading was proven in a failed
study designed to determine how much genetic markers were related to
homosexuality.

5)
The fact that the more the number of gay maternal uncles an offspring
has, the greater his chances of being a homosexual is interesting.
Solution:
By using the word interestingly the author is trying to draw the readers
attention to the fact that the connection to maternal loading was discovered
even though the scientists were looking for something else. The only option
that expresses this is 1.
Option 2 is wrong as there is no real evidence disproving maternal loading.
Option 3 is wrong as the passage states that maternal loading is proven even in
a full genome scan.
Option 4 is wrong as it calls the genome scan study failed when clearly, it is
a success.
Option 5 is wrong as even though it is an interesting fact, that is not the reason
why the author has used that word.
Hence, the correct answer is option 1.


6.
3
Marks
Which among the following can be a result of reduced H-Y antigen
production?
A. Not enough masculinisation of the brain of the male foetus.
B. Production of excessive feminine hormones in a male foetus.
C. A high probability of homosexuality in the male foetus.
D. An ability to fight H-Y antibodies.

1) A only

2) B only

3) A and C

4) A and D

5) All of the above
Solution:
Statement A is correct because it is stated in the passage that Successive male
fetuses are then attacked by H-Y antibodies which somehow decrease the
ability of H-Y antigens to perform their usual function in brain
masculinisation.
Option C is correct as an inability to perform brain masculinisation can lead to
a greater chance of homosexuality in the foetus.
Option B is wrong as there is no evidence in the passage to prove it.
Option D is wrong as the passage says nothing about the ability of a male
foetus to fight H-Y antibodies.
Hence, the correct answer is option 3.


7.
3
Marks
The word most likely to replace preponderance in the passage is:

1) Unbelievable

2) Thoughtfulness

3) Timeliness

4) Predominance

5) Minority
Solution:
The correct answer is 4. Preponderance means superiority in numbers.
The word that comes closest in meaning is predominance. Unbelievable means
incredible, therefore option 1 is incorrect.
Preponderance is not related to thoughtfulness or timeliness, making options 2
and 3 incorrect.
Option 5 is incorrect as minority is an antonym.
Hence, the correct answer is option 4.


8.
3
Marks
The option that best summarizes this passage is:

1)
In 1993, Dean Hamer and his colleagues discovered maternal loading.
Hu built upon these findings, while Mustanski confirmed them. Other
factors, such as the mothers genetic makeup and the number of siblings
also contributed.

2)
Genes play a large role in sexual orientation of a male foetus. Some
contributing factors include a dominance of male gay cousins and uncles
on the mothers side, a few genetic markers, and older brothers. Also, H-
Y antigens are responsible for brain masculinization.

3)
Genetic factors have been proven to be responsible for homosexuality.
The most repetitive finding has been that of maternal loading, which can
therefore be credited as a major factor. Besides it, there are others like
genetic markers, skewing of X-chromosome in a mother and the
presence of older brothers.

4)
A male foetus can be born homosexual if and only if theres a heavy
presence of gay men among his maternal uncles and cousins.

5)
Many genetic factors contribute towards sexual orientation, including
maternal loading, markers on the X-Chromosome at Xq28, along with a
few other genes, extreme skewing of X-chromosome inactivation and
having older brothers.
Solution:
Option 1 is incorrect since who conducted those experiments isnt as important
as the results.
Option 2 is wrong as it is vague about the main points and randomly talks
about H-Y antigens.
Option 3 is wrong as the passage never mentions anything about how maternal
loading is a major factor.
Option 4 is wrong as it incorrect mentions only one point.
Option 5 succinctly paraphrases the key points of the passage, which are not
the studies, but the results. It also mentions the many genetic factors that affect
homosexuality, as mentioned in the passage.
Hence, the correct answer is option 5.


9.
3
Marks
In the following, a sentence is fragmented into four parts. Choose for your
answer the fragment that carries the error. Ignore punctuation errors if any.
Despite his youth, (1) he is someone (2) who I respect (3) a lot.(4)

1) 1

2) 2

3) 3

4) 4

5) No error
Solution:
There is often an error in the usage of who and whom. The basic difference
between the two pronouns is that who is used as the subject of the verb and
whom is used as the object of the verb. In the given sentence, the subject is I
and the highlighted word refers to the object of the verb respect I respect him.
Therefore, whom should be used.
Hence, the correct answer is option 3.


10.
3
Marks
Each of the following questions has a sentence/paragraph with two
underlined words that do not make sense. Choose the most appropriate
replacement for the words from the options given.
In texts that are written according to this view, the gerontological alchemical
symbols, diagrams, and textual imagery of late alchemical works typically contain
multiple layers of meanings, allegories, and references to other equally cryptic
works; and must be laboriously decoded in order to discover their positron
meaning.

1) cryptic, inferred

2) cryptic, true

3) ambiguous, phaneritic

4) apocryphal, esoteric

5) perplexing, plain
Solution:
Cryptic means puzzling or based on a code.
Ambiguous is vague, by accident or intent.
Apocryphal means of doubtful authorship or authenticity.
Perplexing means complicated or confusing.
The phrase, other equally cryptic works in the latter part of the sentence gives
us a clue that cryptic would be the correct choice for the first blank. However,
since the other words for the first blank among the given options have similar
meanings we will consider all the words to be suitable and look for our answer
from the second blank.
Phaneritic is related to rocks that have visible crystals. It is completely
unrelated to the context. This eliminates option 3.
Esoteric means understood only by a select few.
Since the symbols, diagrams and textual imagery contains multiple layers of
meanings , allegories and references to other equally cryptic works, they must be
laboriously decoded in order to discover their real or true meaning. Thus the
second blank cannot have a meaning that is inferred, or understood by only a
select few. Eliminate options 1 and 4.
Moreover, the word plain implies that the meaning is already clear and
straightforward - i.e. it would not have to be decoded. This eliminates option 5.
Hence, the correct answer is option 2.


11.
3
Marks
Select the option which is having similar analogy vis-a-vis the analogy given in
the question.
Seductive: Tempting ::

1) Irascible : Irritable

2) Intense: Aloof

3) Shady: Righteous

4) Vivid: Exalted

5) Shrewd: Rampant
Solution:
Seductive is a synonym of tempting.
Options 2, 3, 4 and 5 do not share the same relationship as the key pair. The
words in option 3 can be termed as antonyms.
Option 1 is the most logical choice as the two words in the option also share a
synonymous relationship.
Hence, the correct answer is option 1.


12.
3
Marks
In the following, a sentence is fragmented into four parts. Choose for your
answer the fragment that carries the error. Ignore punctuation errors if any.
Those who live in (1) reasonable happy surroundings (2) sustain positive
feelings (3) and remain happy.(4)

1) 1

2) 2

3) 3

4) 4

5) No Error
Solution:
The adjective reasonable has been incorrectly used in place of the adverb
reasonably.
Hence, the correct answer is option 2.


13.
3
Marks
Answer the following questions based on the information given below.
Which of the following words means that a person feels worried and is slightly
afraid of something?

1) concerned

2) nervous

3) overwrought

4) uneasy

5) destressed
Solution:
Concerned means feeling concern about somebody.
Nervous is to feel worried and slightly afraid.
Overwrought means worked up; excessively excited or agitated.
Uneasy means uncomfortable.
destressed means having no tension.
Hence, the correct answer is option 2.


14.
3
Marks
Select the odd man out from the given alternatives.


1) Perspicacity

2) Acumen

3) Discernment

4) Humour

5) Acuity
Solution:
Perspicacity, acumen, discernment and acuity all mean the ability to understand
and reason. The odd one out is humour because although it refers to a mental
ability it is not directly related to understanding.
Hence, the correct answer is option 4.


15.
3
Marks
Answer the following question based on the information given below.
Which of the following is a synonym of 'storm'?

1) Hera

2) Squall

3) Torpedo

4) Stroma

5) Croma
Solution:
Squall is a sudden, violent gust of wind, often accompanied by rain, snow, or
sleet.
The other words either do not exist or are unrelated to the meaning of storm.
Hence, the correct answer is option 2.

Group Question


Fill in the blanks in the passage with the most appropriate set of words from the options
for each blank.


Instructional Design is the practice of ___(1)___ the effectiveness, efficiency and
appeal of instruction and other learning experiences. The process consists broadly of
determining the current state and needs of the ___(2)___, defining the end
___(3)___of instruction, and creating some "intervention" to assist in the transition.
The outcome of this instruction may be directly observable and ___(4)___ measured
or completely hidden and ___(5)___.


16.
3
Marks
(1)

1) making

2) motivating

3) maximizing

4) contracting

5) contradicting
Solution:
The first blank requires a word that fits in the context of increasing the
effectiveness, efficiency and appeal of instruction. Making would require an
object making something more..... which is missing. This eliminates option
1.
Motivating, contracting and contradicting are contextually inappropriate.
Maximize means to increase to the greatest possible amount or degree and
is contextually appropriate.
Hence, the correct answer is option 3.


17.
3
Marks
(2)

1) learner

2) people

3) designer

4) populace

5) psychologist
Solution:
The passage talks about instructional design as a learning experience.
Therefore, the process has to determine the state and needs of the learner.
The other options do not fit this context.
Hence, the correct answer is option 1.


18.
3
Marks
(3)

1) destination

2) design

3) motivation

4) cognition

5) goal
Solution:
The passage talks about defining the end ___ of instruction. Therefore, the
blank requires a word that means purpose or target or objective.
Goal means the aim or object towards which an endeavour is directed, and
fits this context. The other options are contextually inappropriate.
Hence, the correct answer is option 5.


19.
3
Marks
(4)

1) directly

2) scientifically

3) physical

4) accurate

5) exact
Solution:
The passage talks about the outcome being directly observable and measured.
The blank requires a word that describes the way the outcome is measured. For
an outcome to be considered valid, its measurement must be scientific,
meaning it must be systematic or accurate in the manner of an exact science.
Directly in option 1 is an unnecessary repetition.
Since the blank describes the verb measured, the answer has to be an adverb.
Options 3, 4 and 5 are adjectives, and are therefore grammatically incorrect.
Hence, the correct answer is option 2.


20.
3
Marks
(5)

1) visible

2) conspicuous

3) distinct

4) assumed

5) apparent
Solution:
The last blank is in the context of an outcome being completely hidden.
According to the passage, The outcome of this instruction may be directly
observable and ... measured OR completely hidden and ___. The or in the
statement indicates that the blank has to contain a word that is the opposite of
something that is directly observed and measured, and it has to be in
consonance with being hidden.
Assumed, meaning supposed or taken for granted is the correct contrast to
something that is directly observed and measured, and therefore hidden. The
remaining words are antonyms of assumed and are therefore contextually
incorrect.
Hence, the correct answer is option 4.


21.
3
Marks
In each of the following sentences, part or the entire sentence is underlined. The
answer-choices offer four ways of phrasing the underlined part. If you think the
original sentence is better than the alternatives, choose 1 which merely repeats
the underlined part; otherwise choose one of the alternatives.
Written in the year 1971, N.S. Pauls On the warfront was the cause of a
sensational nationwide controversy and he had to remain underground for two
years.

1)
N.S. Pauls On the warfront was the cause of a sensational nationwide
controversy and

2)
N.S. Paul and his book, On the warfront were the causes of a sensational
nationwide controversy and

3)
N.S. Pauls On the warfront was the cause of a such a controversial
nationwide sensation that

4)
On the warfront written by N.S. Paul was the cause of a nationwide
controversy so sensational that

5)
N.S. Pauls On the warfront was the cause of a nationwide controversy so
sensational that
Solution:
Since the sentence begins with a modifier, the noun that the modifier qualifies
should be the subject of the sentence. In this case, it is the book On the warfront
and not the author. Option 2 gets eliminated.
Option 4 repeats the verb written, hence can be eliminated.
Option 3 changes the sentence construction by exchanging the words
controversy and sensational.
Between options 1 and 5, option 5 is a better fit as it correctly uses the
construction sothat and it is more fitting than option 1.
Hence, the correct answer is option 5.


22.
3
Marks
In each of the following sentences, part or the entire sentence is underlined. The
answer-choices offer four ways of phrasing the underlined part. If you think the
original sentence is better than the alternatives, choose 1 which merely repeats
the underlined part; otherwise choose one of the alternatives.
On Indian pitches, batsmen play spin with far more ease than most European
pitches.

1) than most European pitches.

2) than on most European pitches.

3) than they do on most European pitches.

4) than they play on most European pitches.

5) than European pitches mostly.
Solution:
The correct option should have a grammatically correct comparison between the
two actions separated by than. The first half of the sentence has an action of
playing which is being compared. This action is not represented in options 1, 2
and 5. Hence, these can be eliminated.
Between options 3 and 4, option 3 is better as it does not repeat the verb play
and replaces it with do, hence making the sentence more refined.
Hence, the correct answer is option 3.

Group Question


A passage is followed by questions pertaining to the passage. Read the passage and
answer the questions. Choose the most appropriate answer.


In psychiatry, the term neologism is used to describe the use of words that only have
meaning to the person who uses them, independent of their common meaning. This is
considered normal in children, but a symptom of thought disorder (indicative of a
psychotic mental illness, such as schizophrenia) in adults. Use of neologisms may
also be related to aphasia acquired after brain damage resulting from a stroke or head
injury.

Neologisms are often created by combining existing words (compound noun or
adjective) or by giving words new and unique suffixes or prefixes. Portmanteaux are
combined words that begin to be used commonly. Neologisms also can be created
through abbreviation or acronym, by intentionally rhyming with existing words or
simply through playing with sounds.

Neologisms often become popular through memetics, by way of mass media, the
Internet, and word of mouth, including academic discourse in many fields renowned
for their use of distinctive jargon, and often become accepted parts of the language.
Other times, however, they disappear from common use just as readily as they
appeared. Whether a neologism continues as part of the language depends on many
factors, probably the most important of which is acceptance by the public. It is
unusual, however, for a word to enter common use if it does not resemble another
word or words in an identifiable way. When a word or phrase is no longer "new", it is
no longer a neologism. Neologisms may take decades to become "old", however.
Opinions differ on exactly how old a word must be to cease being considered a
neologism.


23.
3
Marks
Which of the following statements, based on the passage, is most likely to be
true?

1)
A neologist is one who proposes either a new doctrine or a new
interpretation of source material such as religious texts.

2) Neologisms might lose their novelty when more people start using them.

3)
Aphasia is the reason why many words from the English language have
now been forgotten.

4)
Academic fields with their own distinctive jargon particularly look
forward to neologisms.

5) Both 1 and 2
Solution:
Option 1 is ruled out since this passage is not related to the other meanings or
allusions to the term neologism. This passage is concerned only with its
reference to vocabulary.
Option 3 is also ruled out as this is an exaggerated sentence that has no basis
in the passage.
Option 4 is also ruled out as this can also not be inferred from the information
in the passage.
Option 2 is mentioned in the passage: When a word or phrase is no longer
"new", it is no longer a neologism. Neologisms may take decades to become
"old".
Hence, the correct answer is option 2.


24.
3
Marks
Choose the word in the passage that means a word formed by blending sounds
from two or more distinct words and combining their meanings.

1) Memetics

2) Acronym

3) Portmanteaux

4) Jargon

5) Suffix
Solution:
Portmanteaux refers to a word formed by blending sounds from two or more
distinct words and combining their meanings. For eg: brunch, from
breakfast and lunch, or Spanglish, referring to speaking a mix of both
Spanish and English spoken by bilingual people.
Hence, the correct answer is option 3.


25.
3
Marks
Which of the following is not untrue in the context of the passage?
I. Words can have more than one meaning.
II. Adding a suffix or prefix to a word is called neologism.


1) Only I

2) Only II

3) Neither I nor II

4) Both I and II

5) Either I or II
Solution:
Option 1 is correct because it is mentioned in the passage: the use of words
that only have meaning to the person who uses them, independent of their
common meaning.
Option 2 is incorrect because it isnt the adding of an existing suffix or prefix
that makes a neologism, it is the adding of anew or unique suffix or prefix that
leads to a neologism.
Hence, the correct answer is option 1.


26.
3
Marks
Mark the correct option, which puts the parts of the sentence in right order:
A. Infosys sent an official communique
B. Following the bank's decision
C. To be with Deutsche Network Services
D. To the 150 employees giving them
E. The option to leave the company
F. For much higher salaries


1) ABCDEF

2) BADECF

3) EFCDBA

4) ABDFEC

5) CFDEBA
Solution:
On reading the options one can clearly decipher the sentence to mean that 150
employees of Infosys have been offered much higher salaries by Deutsche
Network Services. Keeping this in mind the ADECF link is clear.
In A, Infosys sends a communique to the 150 employees mentioned in D. The
communique gives these 150 employees the option to leave the company-
mentioned in E- and work for Deutsche Network (mentioned in C) for much
higher salaries mentioned in F.
Statement B can only be the introductory statement since it cannot be
accomodated anywhere else.
Hence, the correct answer is option 2.


27.
3
Marks
Mark the correct option, which puts the parts of the sentence in right order:
A. To connect to it
B. The more it made sense for other networks
C. Over the past 15 years and the bigger it got
D. As it has grown at an astonishing rate
E. A network of networks
F. Internet has become


1) FEDCBA

2) ABCEFD

3) CABDFE

4) EBADFC

5) AFEDCB
Solution:
F is obviously the introductory statement since it introduces the topic being
discussed in the paragraph- which is the internet. We can eliminate options 2,3,4
and 5.
FE is a pair since has become in F can only combine with a network in order
to frame a grammatically correct sentence.
DC is a pair since D mentions the internet's astonishing growth and C mentiones
the timeframe in which it has taken place- 15 years.
BA is also a pair. For other networks in B can only connect grammatically and
logically with to connect to it mentioned in A.
Hence, the correct answer is option 1.


28.
3
Marks
In the following, a sentence is fragmented into four parts. Choose for your
answer the fragment that carries the error. Ignore punctuation errors if any.
It was challenging (1) to study while working, (2) but she was resolved (3) to
take up the challenge.(4)

1) 1

2) 2

3) 3

4) 4

5) No error
Solution:
The sentence is grammatically correct.
Hence, the correct answer is option 5.


29.
3
Marks
Fill in the blanks in the passage with the most appropriate set of words from the
options for each blank.
If we watched the lobster _____ its native element, we should see it climbing
actively the submerged rocks, _____ which it delights to live, by means of its
strong legs; or swimming by powerful strokes of its great tail, the appendages
_____ whose sixth joint are spread out _____ a broad fan-like propeller: seize it,
and it will show you that its great claws are no mean weapons of offence; suspend
a piece of carrion _____ its haunts, and it will greedily devour it, tearing and
crushing the flesh _____ means of its multitudinous jaws.

1) in, among, of, into, among, by

2) from, through, of, into, among, by

3) in, among, from, in, among, by

4) from, among, of, into, by, by

5) in, among, of, into, among, with
Solution:
One studies a species in its native element. Therefore, options 2 and 4 can be
eliminated.
Option 3 is incorrect as the phrase described- the appendages of whose sixth
joint are spread out. The preposition from would be incorrect for this blank.
The use of by is correct for the last blank- ...crushing the flesh by means of ...
This eliminates option 5.
Option 1 uses the appropriate prepositions.
Hence, the correct answer is option 1.


30.
3
Marks
This question consists of two capitalized words that have a certain relationship to
each other, followed by a certain pair of words. Choose the pair that is
RELATED to each other in the same way as the capitalized pair.
SEX : GENDER ::

1) Boy : Girl

2) Child : Children

3) Read : Literacy


4) Population : History

5) Genus : Genera
Solution:
Sex (male/female) determines the gender (masculine/feminine) of a person.
Literacy is determined by ability to read
Children is the plural of Child.
Boy and girl merely show different sexes.
History is not determined by population.
Genera is the plural of genus.
Hence, the correct answer is option 3.

31.
In the following, a sentence is fragmented into four parts. Choose for your answer
the fragment that carries the error. Ignore punctuation errors if any.
Economic reforms brought foreign competition,(1) led to the privatisation of certain
public sector industries, (2) open up sectors hitherto reserved for the public sector (3)
and led to an expansion in the production of fast-moving consumer goods.(4)

1) 1

2) 2

3) 3

4) 4

5) No error
Solution:
The verbs in fragments 1, 2 and 4 are in the past tense. Therefore, even fragment 3
should be in the past tense with opened.
Hence, the correct answer is option 3.

32.
In the following, a sentence is fragmented into four parts. Choose for your answer
the fragment that carries the error. Ignore punctuation errors if any.
Trekking in the monsoons (1) sounds like a lot of fun (2), but look after (3) landslides
and mudflows.(4)

1) 1

2) 2

3) 3

4) 4

5) No error
Solution:
The verb phrase look after means to take care of and does not fit the context of
the given sentence.
The use of the word but in the given sentence indicates a contrary idea- it can be
fun, but ... has to convey something that is NOT fun, something dangerous. Look
out for means watch out for; be concerned about. Look out for landslides and
mudflows fits the context.
Hence, the correct answer is option 3.

You might also like